Decaimiento exponencial del propagador de Feynman fuera del cono de luz

En el capítulo tres (I.3) de Teoría cuántica de campos en pocas palabras de A. Zee , el autor deriva el propagador de Feynman para un campo escalar:

D ( X ) = d 4 k ( 2 π ) 4 mi i k X k 2 metro 2 + i ϵ = i d 3 k ( 2 π ) 3 2 ω k [ mi i ( ω k t k X ) θ ( t ) + mi i ( ω k t k X ) θ ( t ) ]
dónde ω k = k 2 + metro 2 .

Sin trabajar a través de la k integral, el comportamiento del propagador para eventos dentro y fuera del cono de luz se puede analizar aproximadamente (o eso dice el texto): para eventos similares al tiempo en el futuro cono, por ejemplo, X = ( t , X = 0 ) , con t > 0 , el propagador es una suma de ondas planas

D ( t , 0 ) = i d 3 k ( 2 π ) 3 2 ω k mi i ω k t
Del mismo modo, para eventos similares al tiempo en el cono pasado ( t < 0 ) el propagador es una suma de ondas planas con fase opuesta.

Ahora, para eventos similares al espacio, por ejemplo, X = ( 0 , X ) , después de interpretar θ ( 0 ) = 1 2 y observar el propagador permite el intercambio k k , obtenemos

D ( 0 , X ) = i d 3 k ( 2 π ) 3 2 k 2 + metro 2 mi i k X

Luego, el autor afirma que "... el corte de la raíz cuadrada a partir de ± i metro conduce a un decaimiento exponencial mi metro | X | , como cabría esperar. Se deja al lector verificar esto como un problema posterior.

La pregunta es: ¿cómo puedo ver que lo anterior es cierto, sin pasar por el k ¿integral?

En segundo lugar, ¿qué significa "el corte de raíz cuadrada a partir de ± i metro "¿quieres decir? Sé que uno debe proporcionar la raíz cuadrada compleja con un corte de rama, pero dicho corte de rama debe ser un rayo completo del plano, no solo un segmento.

He intentado pasar por la integral; girando el k de modo que X puntos a lo largo del k 3 dirección y cambiando a coordenadas esféricas ( k = | k | , X = | X | ) la integral se convierte en:

D ( 0 , X ) = i 0 d k 0 π d θ 0 2 π d φ ( k 2 pecado θ mi i k X porque θ ( 2 π ) 3 2 k 2 + metro 2 ) = i 0 d k 0 π d θ ( k 2 pecado θ mi i k X porque θ ( 2 π ) 2 2 k 2 + metro 2 ) = i ( 2 π ) 2 0 d k ( k 2 i X k 2 + metro 2 ) mi i k X mi i k X = i ( 2 π ) 2 0 d k k pecado k X X k 2 + metro 2 1 | X |
Que no es el resultado deseado.

¿Tu pregunta es por qué esperamos mi metro X a priori o por qué esperamos obtener eso de esa integral en particular?
Se explica bien en las conferencias QFT de Tong en damtp.cam.ac.uk/user/tong/qft.html . Escribiría la respuesta si recordara, no fue difícil de seguir. Creo que está en la lección 2, pero no puedo estar seguro. Es un poco impactante cuando uno lo ve por primera vez, uno pensaría que debería ser 0 para que c no se exceda, pero la caída exponencial está lo suficientemente cerca. Si reemplaza k= +/- im, verá que exp(-ikx) se convierte en exp(-m|x|), pero (y demasiado perezoso para razonar exactamente ahora) necesita integrar alrededor del infinito y alrededor del corte de las singularidades de una manera determinada (arriba/abajo o al revés). ver pinzas
@Aaron ¿Por qué esperaríamos eso de esa integral en particular?
@BobBee De las notas de D. Tong: "La función D ( X y ) se llama propagador. Para una separación espacial ( X y ) 2 < 0 , se puede demostrar que D ( X y ) se descompone como D ( X y ) mi metro | X y | "
Tal vez fue en las video conferencias reales, estaba convencido. Nunca vi sus notas de clase, pero si mal no recuerdo, mostró cómo derivarlo por la integral alrededor de las singularidades y en el infinito. Gracias
@BobBee Es en la conferencia cinco que D. Tong presenta propagadores; describe en detalle los contornos de integración necesarios para escribir el propagador con funciones de Heaviside en el Feynman i ϵ pero no entra en los cálculos requeridos para mostrar que decae exponencialmente para eventos similares al espacio.

Respuestas (2)

Consulte el artículo de Wikipedia sobre el propagador de Feynman. Su forma en el espacio real es:

GRAMO F ( X , y ) = { 1 4 π d ( τ 2 ) + metro 8 π τ H 1 ( 2 ) ( metro τ ) τ 2 0 i metro 4 π 2 | τ | k 1 ( metro | τ | ) τ 2 < 0 ,
dónde τ 2 ( X 0 y 0 ) 2 ( X y ) 2 , H 1 ( 2 ) es una función de Hankel y k 1 es una función de Bessel modificada del segundo tipo . El resultado deseado se sigue directamente de las propiedades asintóticas de k 1 para grandes argumentos.

Si el resultado deseado es explorar las propiedades de la integral, entonces puede encontrar una respuesta investigando las representaciones integrales de las funciones de Bessel modificadas .

Si está de acuerdo con una prueba de que la función de Bessel modificada es parte de un propagador, aunque no particularmente del propagador de Feynman, y no le preocupa la función delta del cono de luz, se deduce de la continuación analítica de la función de verde euclidiana 4-d .

Lo que haré no es matemáticamente válido, pero creo que funciona un poco como la intuición física:

mi i k X k 2 k i metro k + i metro metro 2 2 i metro mi i k X k + i metro
Usando la generalización del cálculo fraccionario para la fórmula integral de Cauchy:
metro 2 2 i metro 2 π i Γ ( 1 / 2 ) 1 2 mi i k X k 1 2 metro 2 2 i metro 2 π i i π X mi metro X
... bueno... tengo un 1 / X pero tiene la exponencial...